aboutsummaryrefslogtreecommitdiff
diff options
context:
space:
mode:
authorAdrian Kummerlaender2018-03-01 20:30:48 +0100
committerAdrian Kummerlaender2018-03-01 20:30:48 +0100
commitd8181f09290e69043185ed63e4ad6ab8e74869ab (patch)
treef54e024f0148278ab14ccb2e2420b499ed835614
parentb69625a5fb01e835a754aad84709ae1de3135b69 (diff)
downloadmath_reference_sheets-d8181f09290e69043185ed63e4ad6ab8e74869ab.tar
math_reference_sheets-d8181f09290e69043185ed63e4ad6ab8e74869ab.tar.gz
math_reference_sheets-d8181f09290e69043185ed63e4ad6ab8e74869ab.tar.bz2
math_reference_sheets-d8181f09290e69043185ed63e4ad6ab8e74869ab.tar.lz
math_reference_sheets-d8181f09290e69043185ed63e4ad6ab8e74869ab.tar.xz
math_reference_sheets-d8181f09290e69043185ed63e4ad6ab8e74869ab.tar.zst
math_reference_sheets-d8181f09290e69043185ed63e4ad6ab8e74869ab.zip
Update non-inline math environment syntax
-rw-r--r--content/analysis.tex16
-rw-r--r--content/analysis_3.tex76
-rw-r--r--content/eaz.tex10
-rw-r--r--content/funktheo.tex62
-rw-r--r--content/graph_theory.tex38
-rw-r--r--content/lineare_algebra.tex18
-rw-r--r--content/markov.tex24
-rw-r--r--content/numerik_1.tex68
-rw-r--r--content/numerik_2.tex46
-rw-r--r--content/numerik_dgl.tex106
10 files changed, 243 insertions, 221 deletions
diff --git a/content/analysis.tex b/content/analysis.tex
index b43b63f..119d0d8 100644
--- a/content/analysis.tex
+++ b/content/analysis.tex
@@ -30,7 +30,7 @@ $\forall K \in \N \exists N_K \in \N \forall n \geq N_K : x_n \geq K \Leftrighta
\subsection*{Beispiele und Hinweise}
-$$e^x = exp(x) = \lim_{n\to \infty} \Big(1 + \frac{x}{n}\Big)^n \text{ insb. } e = \lim_{n\to \infty} \Big(1 + \frac{1}{n}\Big)^n$$
+\[ e^x = exp(x) = \lim_{n\to \infty} \Big(1 + \frac{x}{n}\Big)^n \text{ insb. } e = \lim_{n\to \infty} \Big(1 + \frac{1}{n}\Big)^n \]
Zur Bestimmung von Folgen Grenzwerten kann auch L'Hospital herangezogen werden.
@@ -393,10 +393,10 @@ $\forall \epsilon > 0 \exists N_\epsilon \in \N \forall n \geq N_\epsilon : ||x_
\subsubsection*{$p$-Norm}
\vspace{-4mm}
-$$|x|_p := \begin{cases}
+\[ |x|_p := \begin{cases}
(\sum_{k=1}^m |x_k|^p )^{\frac{1}{p}} & 1 \leq p < \infty \\
\max_{1\leq k \leq m} |x_k| & p = \infty
-\end{cases}$$
+\end{cases} \]
\subsubsection*{Hölder-Ungleichung}
@@ -817,20 +817,20 @@ Sei $\gamma \in C([a, b], \R^m)$ stückweise $C^1$, $\Gamma = \gamma([a, b])$.
Sei reelles $f \in C(\Gamma, \R)$ gegeben:
\vspace*{-5mm}
-$$\int_\Gamma f d\gamma = \int_\Gamma f(x) d\gamma := \int_a^b f(\gamma(t)) | \gamma'(t) |_2 dt$$
+\[ \int_\Gamma f d\gamma = \int_\Gamma f(x) d\gamma := \int_a^b f(\gamma(t)) | \gamma'(t) |_2 dt \]
\subsubsection*{Kurvenintegral zweiter Art}
Sei vektorwertiges $F \in C(\Gamma, \R^m)$ gegeben:
\vspace*{-5mm}
-$$\int_\Gamma F \cdot dx = \int_\Gamma F(x) \cdot dx := \int_a^b (F(\gamma(t))|\gamma'(t)) dt$$
+\[ \int_\Gamma F \cdot dx = \int_\Gamma F(x) \cdot dx := \int_a^b (F(\gamma(t))|\gamma'(t)) dt \]
\subsubsection*{Wegunabhängigkeit}
Sei $D \subseteq \R^m$ offen, dann ist $F \in C(D, \R^m)$ wegunabhängig auf $D$, wenn für alle stückweisen $C^1$-Kurven $\gamma_1, \gamma_2 \in C([a, b], \R^m)$ in $D$ mit gleichem Anfangs- und Endpunkt gilt:
-$$\int_{\Gamma_1} F \cdot dx = \int_{\Gamma_2} F \cdot dx$$
+\[ \int_{\Gamma_1} F \cdot dx = \int_{\Gamma_2} F \cdot dx \]
Ein $\phi \in C^1(D, \R)$ heißt Potential von $F$ auf $D$, wenn $\nabla\phi = F$ auf $D$. $F$ ist dann Gradientenfeld.
@@ -915,7 +915,7 @@ Jedes Anfangswertproblem $k$-ter Ordnung lässt sich in ein Problem 1. Ordnung u
Beispielsweise: Das Problem 2. Ordnung $u''(t)=h(t)-u(t)+u'(t)^2$ mit $u(0)=u_0$ und $u'(0)=u_1$ sowie $h \in C(\R, \R)$ wird formuliert als Problem 1. Ordnung:
-$$\begin{pmatrix}u(t)\\u'(t)\end{pmatrix}' = \begin{pmatrix}u'(t)\\u''(t)=h(t)-u(t)+u'(t)^2\end{pmatrix}$$
+\[ \begin{pmatrix}u(t)\\u'(t)\end{pmatrix}' = \begin{pmatrix}u'(t)\\u''(t)=h(t)-u(t)+u'(t)^2\end{pmatrix} \]
Sei $v_0(t):=u(t)$, $v_1(t):=u'(t)$ und $v(t):=\begin{pmatrix}v_0(t)\\v_1(t)\end{pmatrix}$
@@ -938,7 +938,7 @@ Insgesamt also:
Sei $u'(t)=g(t)h(u(t))$ mit $u(t_0)=u_0$ Anfangswertproblem mit $g \in C(\R, \R)$, $h \in C((a, b), \R)$, $u_0 \in (a, b)$ und $h(u_0) \neq 0$. $u$ ist Lösung, wenn $J$ Intervall mit $\forall t \in J : u(t) \in (a, b)$, $u \in C^1(J, \R)$ und $t_0 \in J$.
\vspace*{-5mm}
-$$u \text{ ist Lösung } \Rightarrow \int_{t_0}^t g(s) ds = \int_{u_0}^{u(t)} \frac{1}{h(x)} dx$$
+\[ u \text{ ist Lösung } \Rightarrow \int_{t_0}^t g(s) ds = \int_{u_0}^{u(t)} \frac{1}{h(x)} dx \]
\vspace*{-3mm}
Dies kann manchmal nach $u$ aufgelöst werden.
diff --git a/content/analysis_3.tex b/content/analysis_3.tex
index 9803c81..98420bf 100644
--- a/content/analysis_3.tex
+++ b/content/analysis_3.tex
@@ -1,7 +1,7 @@
% Borel Sigma-Algebra Kürzel
\newcommand{\A}{\mathcal{A}}
\newcommand{\B}{\mathcal{B}}
-\newcommand{\C}{\mathcal{C}}
+\renewcommand{\C}{\mathcal{C}}
\newcommand{\E}{\mathcal{E}}
\newcommand{\F}{\mathcal{F}}
\newcommand{\J}{\mathcal{J}}
@@ -16,7 +16,7 @@
Sei $\B$ ein Mengensystem.
-$$\left(\bigcup_{B\in \B} B \right)^c = \bigcap_{B\in \B} B^c \hspace*{8mm} \left(\bigcap_{B\in \B} \right)^c = \bigcup_{B\in \B} B^c$$
+\[ \left(\bigcup_{B\in \B} B \right)^c = \bigcap_{B\in \B} B^c \hspace*{8mm} \left(\bigcap_{B\in \B} \right)^c = \bigcup_{B\in \B} B^c \]
\subsection*{Mengen-Ring}
@@ -55,7 +55,7 @@ Seien $\A$ eine $\sigma$-Algebra auf $X$, $n \in \N$, $\forall j \in \N : A_j \i
Die durch das nichtleere Mengensystem $\E \subseteq \powerset{X}$ auf $X$ erzeugte $\sigma$-Algebra ist wie folgt definiert:
\vspace*{-4mm}
-$$\sigma(\E) := \bigcap\{ \A \subseteq \powerset{X} | \A \text{ ist } \sigma \text{-Algebra}, \E \subseteq \A \}$$
+\[ \sigma(\E) := \bigcap\{ \A \subseteq \powerset{X} | \A \text{ ist } \sigma \text{-Algebra}, \E \subseteq \A \} \]
Der Erzeuger $\E$ ist hierbei allg. nicht eindeutig.
@@ -80,7 +80,7 @@ $\B_m$ enthält insb. alle offenen und abgeschlossenen Mengen in $\R^m$ sowie de
\subsubsection*{Charakterisierung}
-$$\B_m = \sigma(\{(a, b) | a, b \in \mathbb{Q}^m, a \leq b\})$$
+\[ \B_m = \sigma(\{(a, b) | a, b \in \mathbb{Q}^m, a \leq b\}) \]
Analoges gilt auch für andere Intervalle.
@@ -107,10 +107,10 @@ Ein Wahrscheinlichkeitsmaß erfüllt $\mu(X) = 1$.
Für fest gewählte $\A = \powerset{X}$, $x \in X$ ist ein Wahrscheinlichkeitsmaß für $A \subseteq X$ definiert:
\vspace{-2mm}
-$$\delta_x(A) := \begin{cases}
+\[ \delta_x(A) := \begin{cases}
1 & x \in A \\
0 & x \notin A
-\end{cases}$$
+\end{cases} \]
\subsection*{Zählmaß}
@@ -154,7 +154,7 @@ $\lambda(I) = \lambda_m(I) := (b_1 - a_1) \cdot \hdots \cdot (b_m - a_m)$
\subsection*{Ring der Figuren}
-$$\F_m = \left\{ A = \bigcup_{j=1}^n I_j | I_j \in \J_m, n \in \N \right\}$$
+\[ \F_m = \left\{ A = \bigcup_{j=1}^n I_j | I_j \in \J_m, n \in \N \right\} \]
\subsubsection*{Eigenschaften des Ring der Figuren}
@@ -227,10 +227,10 @@ Weiterhin gilt: $\overline \B_1 = \sigma(\{ [-\infty,a] | a \in \Q \})$
Seien $f_n : X \to \overline \R$ für alle $n \in \N$ $\A-\overline \B_1$-messbar
\vspace{-4mm}
-$$\Rightarrow \sup_{n \in \N} f_n, \inf_{n \in \N} f_n, \varliminf_{n \to \infty} f_n, \varlimsup_{n \to \infty} f_n \ \ \A-\overline \B_1 \text{-messbar}$$
+\[ \Rightarrow \sup_{n \in \N} f_n, \inf_{n \in \N} f_n, \varliminf_{n \to \infty} f_n, \varlimsup_{n \to \infty} f_n \ \ \A-\overline \B_1 \text{-messbar} \]
\vspace{-4mm}
-$$\forall x \in X : \lim_{n \to \infty} f_n(x) \in \overline \R \Rightarrow \lim_{n \to \infty} f_n \text{ ist } \A-\overline \B_1-\text{mb.}$$
+\[ \forall x \in X : \lim_{n \to \infty} f_n(x) \in \overline \R \Rightarrow \lim_{n \to \infty} f_n \text{ ist } \A-\overline \B_1-\text{mb.} \]
$f : [a,b] \to \R$ diffbar. $\Rightarrow f'$ ist $\B([a,b])$-$\B_1$-mb.
@@ -257,7 +257,7 @@ Dann ist auch $|f| : x \mapsto |f(x)|$ $\A$-$\overline\B_+$-mb.
Messbare $f : X \to \R$ heißt einfach, wenn sie nur endlich viele Werte annimmt. Die Normalform von $f$ ist für $f^{-1}(\{y\}) \in \A$ mit $y \in f(X)$ definiert:
\vspace{-2mm}
-$$f = \sum_{y \in f(X)} y \cdot \1_{f^{-1}(\{y\})}$$
+\[ f = \sum_{y \in f(X)} y \cdot \1_{f^{-1}(\{y\})} \]
Sei $f : X \to \overline R$ messbar, dann gelten:
@@ -273,19 +273,19 @@ $f : X \to \overline\R$ ist $\A$-$\overline\B_1$-mb. gdw. einfache Fkt. $f_n : X
\subsection*{Integral für nichtnegative einfache Fkt.}
-$$\int f d\mu = \int_X f(x) d\mu(x) := \sum_{y \in f(X)} y \cdot \mu(f^{-1}(\{y\}))$$
+\[ \int f d\mu = \int_X f(x) d\mu(x) := \sum_{y \in f(X)} y \cdot \mu(f^{-1}(\{y\})) \]
\subsection*{Integral für nichtnegative Funktionen}
Sei $f : X \to [0, \infty]$ $\A$-$\overline\B_+$-mb. und steigende Folge einfacher $f_n \leq f$ mit $\displaystyle\lim_{n \to \infty} f_n(x) = f(x)$ gegeben:
\vspace{-2mm}
-$$\int_X f \ d\mu := \lim_{n \to \infty} \int_X f_n \ d\mu = \sup_{n \in \N} \int_X f_n \ d\mu$$
+\[ \int_X f \ d\mu := \lim_{n \to \infty} \int_X f_n \ d\mu = \sup_{n \in \N} \int_X f_n \ d\mu \]
Grundlegende Integraleigenschaften sind erfüllt.
\vspace{-4mm}
-$$\int_X f \ d\mu = \sup\left\{ \int_X g \ d\mu | g \text{ einfach}, 0 \leq g \leq f \right\}$$
+\[ \int_X f \ d\mu = \sup\left\{ \int_X g \ d\mu | g \text{ einfach}, 0 \leq g \leq f \right\} \]
\subsection*{Monotone Konvergenz}
@@ -303,18 +303,18 @@ Dies gilt nicht ohne Monotonie oder für eine fallende Folge $(f_n)_{n \in \N}$.
Seien $f_j : X \to [0,\infty]$ $\A$-$\overline\B_+$-mb. für $\forall j \in \N$. Dann ist auch $\sum_{j=1}^\infty f_j$ $\A$-$\overline\B_+$-messbar und:
-$$\int_X \sum_{j=1}^\infty f_j(x) d\mu(x) = \sum_{j=1}^\infty \int_X f_j(x) d\mu(x)$$
+\[ \int_X \sum_{j=1}^\infty f_j(x) d\mu(x) = \sum_{j=1}^\infty \int_X f_j(x) d\mu(x) \]
\subsection*{Integral für $\overline\R$-wertige Funktionen}
Sei $f : X \to \overline\R$ eine $\A$-$\overline\B_1$-mb. Funktion. Dann sind auch $f_+$ und $f_-$ mb. $f$ ist Lebesgue-integrierbar, wenn:
\vspace{-4mm}
-$$\int_X f_+(x) d\mu(x) < \infty \text{ und } \int_X f_-(x) d\mu(x) < \infty$$
+\[ \int_X f_+(x) d\mu(x) < \infty \text{ und } \int_X f_-(x) d\mu(x) < \infty \]
Das Lebesgue-Integral ist dann definiert durch:
-$$\int f d\mu = \int_X f_+(x) d\mu(x) - \int_X f_-(x) d\mu(x)$$
+\[ \int f d\mu = \int_X f_+(x) d\mu(x) - \int_X f_-(x) d\mu(x) \]
$\L^1(X,\A,\mu) = \L^1(\mu) = \L^1(X) := \{ f : X \to \R | f \text{ ib.}\}$
@@ -389,12 +389,12 @@ Sei $f : X \to \overline\R$ ib. Dann ist $\{|f|=\infty\}$ eine Nullmenge, $f$ is
Sei $f_n : X \to [0,\infty]$ für alle $n \in \N$ mb. Dann:
\vspace{-2mm}
-$$\int_X \liminf_{n \to \infty} f_n d\mu \leq \liminf_{n \to \infty} \int_X f_n d\mu$$
+\[ \int_X \liminf_{n \to \infty} f_n d\mu \leq \liminf_{n \to \infty} \int_X f_n d\mu \]
Konvergiert $f_n$ f.ü. gegen mb. $f : X \to [0,\infty]$:
\vspace{-2mm}
-$$\int_X f d\mu \leq \liminf_{n \to \infty} \int_X f_n d\mu$$
+\[ \int_X f d\mu \leq \liminf_{n \to \infty} \int_X f_n d\mu \]
\subsection*{Majorisierte Konvergenz (Lebesgue)}
@@ -423,7 +423,7 @@ Sei $M$ metrischer Raum, $t_0 \in M$, $(X,\A,\mu)$ Maßraum und $f : M \times X
Dann ist $\forall t \in M$ die Fkt. $X \to \R; x \mapsto f(t,x)$ ib. und es gilt:
\vspace{-2mm}
-$$\lim_{t \to t_0} \int_X f(t,x) d\mu(x) = \int_X f(t_0,x) d\mu(x)$$
+\[ \lim_{t \to t_0} \int_X f(t,x) d\mu(x) = \int_X f(t_0,x) d\mu(x) \]
\subsection*{Differentiationssatz}
@@ -437,7 +437,7 @@ Sei $U \subseteq \R^k$ offen, $j \in \{1,\cdots,k\}$, $(X,\A,\mu)$ Maßraum und
Dann ist $\forall t \in U$ die Abbildung $x \mapsto \frac{\partial}{\partial t_j} f(t,x)$ integierbar und es existiert die partielle Ableitung:
-$$\frac{\partial}{\partial t_j} \int_X f(t,x) dx = \int_X \frac{\partial f}{\partial t_j} (t,x) dx$$
+\[ \frac{\partial}{\partial t_j} \int_X f(t,x) dx = \int_X \frac{\partial f}{\partial t_j} (t,x) dx \]
\section*{Iterierte Integrale}
@@ -455,7 +455,7 @@ Für $C \subseteq \R^m$ sind die Schnitte definiert:
Für beliebige $C \in \B_m$ gilt:
-$$\lambda_m(C) = \int_{\R^k} \lambda_l(C^x) dx = \int_{\R^l} \lambda_k(C_y) dy$$
+\[ \lambda_m(C) = \int_{\R^k} \lambda_l(C^x) dx = \int_{\R^l} \lambda_k(C_y) dy \]
Daraus folgt:
@@ -488,7 +488,7 @@ Sei $U \subseteq \R^m$ offen, $\phi \in C^1(U,\R^m)$ injektiv und $A \in \B_m$ m
Dann ist $\phi(A^\circ)$ offen, $\phi : A^\circ \to \phi(A^\circ)$ Diffeomorphismus und $\phi(A) \setminus \phi(A^\circ)$ Nullmenge. Weiter:
\begin{enumerate}[label=(\alph*)]
- \item Sei $f: \phi(A) \to [0,\infty]$ messbar. Dann: \vspace{-2mm} $$\int_{\phi(A)} f(y) dy = \int_A f(\phi(x))|\det \phi'(x)| dx$$
+ \item Sei $f: \phi(A) \to [0,\infty]$ messbar. Dann: \vspace{-2mm} \[ \int_{\phi(A)} f(y) dy = \int_A f(\phi(x))|\det \phi'(x)| dx \]
\item Sei $f : \phi(A) \to \overline\R$ messbar. Dann ist $f$ auf $\phi(A)$ ib. gdw. $x \mapsto f(\phi(x))|\det(\phi'(x))|$ auf $A$ integrierbar ist. Es gilt dann auch (a).
\end{enumerate}
@@ -515,7 +515,7 @@ Eine Borelmenge $M \subseteq \R^m$ ist \emph{dünnsinguläre} $C^k$-Hyperfläche
Sei $F : U \to W$ eine Parametrisierung. Dann ist
\vspace{-2mm}
-$$g_F(t) = \transpose{\det(F'(t)}F'(t))$$
+\[ g_F(t) = \transpose{\det(F'(t)}F'(t)) \]
die \emph{Gramsche Determinante} von $F$. Die Matrix $\transpose{F'(t)}F'(t) \in L(\R^{m-1})$ ist sym. und positiv definit.
@@ -546,7 +546,7 @@ $\sqrt{g_F(r,\varphi,\theta)} = r^{m-1} \cos^1(\theta_1) \cdots \cos^{m-2}(\thet
Sei $F : U \to W$ eine Parametrisierung, $M_0 = F(U) \subseteq \R^m$ ein offenes Flächenstück. Sei weiter $f : M_0 \to \overline\R$ messbar und nichtnegativ oder die Funktion $g := f \circ F \sqrt{g_F}$ integrierbar. Dann:
\vspace{-4mm}
-$$\int_{M_0} f d\sigma = \int_{M_0} f(x) d\sigma(x) := \int_U f(F(t))\sqrt{g_F(t)} dt$$
+\[ \int_{M_0} f d\sigma = \int_{M_0} f(x) d\sigma(x) := \int_U f(F(t))\sqrt{g_F(t)} dt \]
\subsubsection*{Oberflächenmaß}
@@ -564,19 +564,19 @@ Maß ist unabhg. der Wahl der Parametrisierung.
Sei $D \subseteq \R^m$ offen und beschränkt mit dünnsingulärem $C^1$-Rand, liege $f \in C(\overline D,\R^m) \cap C_b^1(D,\R^m)$ und $(f|\nu) \in \L^1(\partial D,\sigma)$. Dann:
-$$\int_D div f(x) dx = \int_{\partial D} (f(x)|\nu(x)) d\sigma(x)$$
+\[ \int_D div f(x) dx = \int_{\partial D} (f(x)|\nu(x)) d\sigma(x) \]
Mit $\text{div} f(x) := \text{spur} \ f'(x) = \partial_1 f_1(x) + \cdots + \partial_m f_m(x)$ und $\nu$ ist äußere Einheitsnormale an $\partial D$:
\vspace{-4mm}
-$$\nu(x) = \tfrac{1}{\sqrt{1+|\nabla h(x')|_2^2}} \begin{pmatrix} -\nabla h(x') \\ 1 \end{pmatrix}, \ \ x = (x',x_m)$$
+\[ \nu(x) = \tfrac{1}{\sqrt{1+|\nabla h(x')|_2^2}} \begin{pmatrix} -\nabla h(x') \\ 1 \end{pmatrix}, \ \ x = (x',x_m) \]
\subsection*{Satz von Stokes in $\R^3$}
Für $f \in C^1(D,\R^3)$ ist die Rotation definiert:
\vspace{-4mm}
-$$\text{rot} \ f(x) = \begin{pmatrix}
+\[ \text{rot} \ f(x) = \begin{pmatrix}
\partial_1 \\
\partial_2 \\
\partial_3
@@ -588,23 +588,23 @@ $$\text{rot} \ f(x) = \begin{pmatrix}
\partial_2 f_3(x) - \partial_3 f_2(x) \\
\partial_3 f_1(x) - \partial_1 f_3(x) \\
\partial_1 f_2(x) - \partial_2 f_1(x)
-\end{pmatrix}$$
+\end{pmatrix} \]
Sei $F : U \to D$ $C^2$-Parametrisierung mit offenen, beschränken $U \subseteq \R^2$, $D \subseteq \R^3$. Die äußere Einheitsnormale $n$ an $M = F(U) \subseteq D$ sei:
\vspace{-4mm}
-$$n(F(t)) = \frac{1}{|\partial_1 F(t) \times \partial_2 F(t)|_2} \partial_1 F(t) \times \partial_2 F(t)$$
+\[ n(F(t)) = \frac{1}{|\partial_1 F(t) \times \partial_2 F(t)|_2} \partial_1 F(t) \times \partial_2 F(t) \]
Weiter sei $\partial U$ geschlossene und doppelpunktfreie $C^1$-Kurve mit Parametrisierung $\gamma : (a,b) \to \partial U$ im Gegenuhrzeigersinn s.d. $\partial M$ $C^1$-Kurve mit Parametrisierung $\varphi = F \circ \gamma$ ist.
Dann gilt:
\vspace{-2mm}
-$$\int_M (\text{rot} \ f(x) | n(x)) d\mu(x) = \int_{\partial M} f \cdot \ dx$$
+\[ \int_M (\text{rot} \ f(x) | n(x)) d\mu(x) = \int_{\partial M} f \cdot \ dx \]
Dabei ist das \emph{Kurvenintegral zweiter Art} geg. als:
\vspace{-2mm}
-$$\int_{\partial M} f \cdot dx = \int_a^b (f(\varphi(\tau))|\varphi'(\tau)) d\tau$$
+\[ \int_{\partial M} f \cdot dx = \int_a^b (f(\varphi(\tau))|\varphi'(\tau)) d\tau \]
\section*{Lebesguesche Räume}
@@ -650,30 +650,30 @@ $(L^p(\mu),\|\cdot\|_p)$ ist $\forall p \in [1,\infty]$ normierter Vektorraum.
Sei $\frac{1}{p} + \frac{1}{p'} = 1$ mit:
\vspace{-4mm}
-$$p' = \begin{cases}
+\[ p' = \begin{cases}
\frac{p}{p-1} & p \in (1, \infty) \\
\infty & p = 1 \\
1 & p = \infty
-\end{cases}$$
+\end{cases} \]
Dann liegt für $f \in \L^p(\mu)$, $g \in \L^{p'}(\mu)$ das Produkt $fg \in \L^1(\mu)$ und die Höldersche Ungleichung gilt:
\vspace{-4mm}
-$$\left| \int_X fg d\mu \right| \leq \int_X |fg| d\mu = \|fg\|_1 \leq \|f\|_p \|g\|_{p'}$$
+\[ \left| \int_X fg d\mu \right| \leq \int_X |fg| d\mu = \|fg\|_1 \leq \|f\|_p \|g\|_{p'} \]
\subsection*{Minkowski Ungleichung}
Seien $f, g \in \L^p(\mu)$. Dann gilt $f + g \in \L^p(\mu)$ und:
\vspace{-2mm}
-$$\| f + g \|_p \leq \|f\|_p + \|g\|_p$$
+\[ \| f + g \|_p \leq \|f\|_p + \|g\|_p \]
\subsection*{Konvergenz in $\L^p$}
Sei $\mu(X) < \infty$ und $1 \leq p \leq q \leq \infty$. Dann gilt $\L^q(\mu) \subseteq \L^p(mu)$ und für $f \in L^q(\mu)$:
\vspace{-2mm}
-$$\|f\|_p \leq \mu(X)^{\frac{1}{p} - \frac{1}{q}} \|f\|_q$$
+\[ \|f\|_p \leq \mu(X)^{\frac{1}{p} - \frac{1}{q}} \|f\|_q \]
Die Konvergenz $\| f - f_n \|_p \to 0$ folgt also in diesem Fall aus $\| f - f_n \|_q \to 0$ für $n \to \infty$.
@@ -694,7 +694,7 @@ $L^p(\mu)$ ist ein Banach-, für $p=2$ ein Hilbertraum.
Sei $(X,\A,\mu)$ Maßraum und $f \in L^p(\mu)$. Dann liegt $E = \{ f \in L^p(\mu) | f \text{ ist einfach} \}$ dicht in $L^p(\mu)$, d.h:
\vspace{-4mm}
-$$\forall f \in L^p(\mu), \epsilon > 0 \exists \text{ einf. } g \in L^p(\mu) : \| f - g \|_p \leq \epsilon$$
+\[ \forall f \in L^p(\mu), \epsilon > 0 \exists \text{ einf. } g \in L^p(\mu) : \| f - g \|_p \leq \epsilon \]
\section*{Komplexe Integrale}
@@ -706,4 +706,4 @@ Für die Integrierbarkeit von mb. $f : X \to \mathbb{C}$ gilt:
$|f| : X \to [0,\infty)$ ib. $\Leftrightarrow \text{Re} f, \text{Im} f : X \to \R$ ib.
-$$\int_X f d\mu := \int_X \text{Re} f d\mu + i \int_X \text{Im} f d\mu$$
+\[ \int_X f d\mu := \int_X \text{Re} f d\mu + i \int_X \text{Im} f d\mu \]
diff --git a/content/eaz.tex b/content/eaz.tex
index 010c4a8..ba08f7d 100644
--- a/content/eaz.tex
+++ b/content/eaz.tex
@@ -488,11 +488,11 @@ Das Bild von $F^\times \to F^\times, b \mapsto b^2$ ist Quadratmenge.
Sei $p \geq 3$ Primzahl. Für $a \in \Z$ ist def.:
\vspace*{-2mm}
-$$\legendre{a}{p} = \begin{cases}
+\[ \legendre{a}{p} = \begin{cases}
0 & p | a \\
1 & \exists x \in \Z \setminus p\Z : a \equiv x^2 \ (mod \ p) \\
-1 & \text{sonst}
-\end{cases}$$
+\end{cases} \]
$\legendre{a}{p}$ ist das \emph{Legendre-Symbol} von $a$ modulo $p$.
@@ -503,10 +503,10 @@ $\legendre{a}{p}$ ist das \emph{Legendre-Symbol} von $a$ modulo $p$.
Sei $a \in \Z, m, n \in \Z : a=mn, p \in \Primes$:
\vspace*{-2mm}
-$$\begin{array}{ll}
+\[ \begin{array}{ll}
\legendre{a}{p} = \legendre{a-p}{p} & \legendre{m \cdot n}{p} = \legendre{m}{p}\legendre{n}{p} \\
\legendre{2}{p} = (-1)^{\frac{p^2-1}{8}} & \legendre{-1}{p} = (-1)^{\frac{p-1}{2}}
-\end{array}$$
+\end{array} \]
Sei $l, p \in \Primes$ mit $l, p \neq 2$:
@@ -578,7 +578,7 @@ Sei $R$ ein solcher Hauptidealring. Dann:
Seien $R$ Hauptidealring, $r, s \in R$ teilerfremd (d.h. $1=rx+sy$ für geeignete $x, y \in R$). Dann gilt für Ideale $I = Rr, J = Rs$ der Chinesische Restsatz s.d.:
\vspace*{-2mm}
-$$R/(Rrs) \cong R/(Rr) \times R/(Rs)$$
+\[ R/(Rrs) \cong R/(Rr) \times R/(Rs) \]
$\forall a, b \in R \exists x \in R : x \equiv a \ (mod \ Rr) \land x \equiv b \ (mod \ Rs)$
diff --git a/content/funktheo.tex b/content/funktheo.tex
index b47c2b0..22c2723 100644
--- a/content/funktheo.tex
+++ b/content/funktheo.tex
@@ -5,7 +5,7 @@ $\C = \{ z = x+iy | x,y \in \R \}$
$\C$ wird via $z = x + iy \mapsto (x,y)$ mit $\R^2$ identifiziert.
\vspace*{-4mm}
-$$z \cdot w = \begin{pmatrix} x & -y \\ y & x\end{pmatrix} \begin{pmatrix} u \\ v\end{pmatrix} = \begin{pmatrix} r & 0 \\ 0 & r\end{pmatrix} \begin{pmatrix} \frac{x}{r} & -\frac{y}{r} \\ \frac{y}{r} & \frac{x}{r}\end{pmatrix} \begin{pmatrix} u \\ v\end{pmatrix}$$
+\[ z \cdot w = \begin{pmatrix} x & -y \\ y & x\end{pmatrix} \begin{pmatrix} u \\ v\end{pmatrix} = \begin{pmatrix} r & 0 \\ 0 & r\end{pmatrix} \begin{pmatrix} \frac{x}{r} & -\frac{y}{r} \\ \frac{y}{r} & \frac{x}{r}\end{pmatrix} \begin{pmatrix} u \\ v\end{pmatrix} \]
wobei $r := \sqrt{x^2 + y^2}$. Es gilt für die orthogonale Matrix $D = \begin{pmatrix} \frac{x}{r} & -\frac{y}{r} \\ \frac{y}{r} & \frac{x}{r}\end{pmatrix}$: $\det D = 1$ d.h. die komplexe Multiplikation ist eine Drehstreckung.
@@ -21,12 +21,12 @@ $\displaystyle\lim_{n \to \infty} z_n = z$ in $\C \iff \displaystyle\lim_{n \to
Für $z = x +iy \in \C \setminus \{0\}$ gilt $z = re^{i\phi}$ mit $r = |z|$ und:
\vspace*{-2mm}
-$$\phi = \arg z := \begin{cases}
+\[ \phi = \arg z := \begin{cases}
\arccos \frac{x}{r} & y > 0 \\
0 & x \in (0,+\infty) \\
-\arccos \frac{x}{r} & y < 0 \\
\pi & z \in (-\infty,0)
-\end{cases}$$
+\end{cases} \]
mit $\phi \in (-\pi, \pi]$. Es gilt für $z = re^{i\phi}, w = se^{i\psi}$:
@@ -37,7 +37,7 @@ $z \cdot w = rse^{i(\phi+\psi)} = |z||w|e^{i(\phi+\psi)}$.
Eine Funktion $f : D \to \C$ ist \emph{komplex differenzierbar} in $z_0 \in D$, wenn:
\vspace*{-4mm}
-$$f'(z_0) := \displaystyle\lim_{z \to z_0, z \in D\setminus\{z_0\}} \frac{f(z)-f(z_0)}{z-z_0} \in \C \text{ existiert.}$$
+\[ f'(z_0) := \displaystyle\lim_{z \to z_0, z \in D\setminus\{z_0\}} \frac{f(z)-f(z_0)}{z-z_0} \in \C \text{ existiert.} \]
Ist $f$ in $\forall z_0 \in D$ komplex differenzierbar, so heißt $f$ \emph{holomorph} auf $D$ mit Ableitung $f' : D \to \C$.
@@ -64,7 +64,7 @@ Polynome $p$ und nichtsinguläre rationale Funktionen aus Polynomen sind auf $\C
Seien $a_k \in \C, k \in \N_0$:
\vspace*{-2mm}
-$$\rho = \frac{1}{\overline\lim_{k\to\infty} \sqrt[k]{|a_k|}} \in [0,+\infty]$$
+\[ \rho = \frac{1}{\overline\lim_{k\to\infty} \sqrt[k]{|a_k|}} \in [0,+\infty] \]
ist der \emph{Konvergenzradius}.
@@ -75,7 +75,7 @@ $f : B(c,\rho) \to \C, z \mapsto \sum_{k=0}^\infty a_k(z-c)^k$.
Diese ist auf $B(c,\rho)$ beliebig oft komplex differenzierbar. Für $n \in \N_0$ hat $f^{(n)}$ den Konvergenzradius $\rho > 0$ und es gilt für $z \in B(c,\rho)$:
\vspace*{-4mm}
-$$f^{(n)}(z) = \sum_{k=n}^\infty k(k-1)\cdots(k-n+1)a_k(z-c)^{k-n}$$
+\[ f^{(n)}(z) = \sum_{k=n}^\infty k(k-1)\cdots(k-n+1)a_k(z-c)^{k-n} \]
Auf diese Weise ergeben sich für $z \in \C$:
@@ -99,15 +99,15 @@ Es sind dann äquivalent:
\begin{enumerate}[label=(\alph*)]
\item $f$ ist in $z$ komplex differenzierbar
\item $f$ ist in $z$ reell differenzierbar und es gelten die \emph{Cauchy-Riemannschen DGL}: \\
- $$\frac{\partial u}{\partial x}(x,y) = \frac{\partial v}{\partial y}(x,y), \frac{\partial u}{\partial y}(x,y) = -\frac{\partial v}{\partial x}(x,y)$$
+ \[ \frac{\partial u}{\partial x}(x,y) = \frac{\partial v}{\partial y}(x,y), \frac{\partial u}{\partial y}(x,y) = -\frac{\partial v}{\partial x}(x,y) \]
\end{enumerate}
$f$ hat in $(x,y) \in D \subseteq \R^2$ die \emph{Jacobimatrix}:
-$$f'(z) = \begin{pmatrix}
+\[ f'(z) = \begin{pmatrix}
\frac{\partial u}{\partial x}(x,y) & \frac{\partial u}{\partial y}(x,y) \\
-\frac{\partial u}{\partial y}(x,y) & \frac{\partial u}{\partial x}(x,y)
-\end{pmatrix}$$
+\end{pmatrix} \]
Entsprechend ist $f(z)=\overline z$ nirgends komplex differenzierbar, $f(z)=|z|^2$ nur in $0$ komplex differenzierbar und $f(z) = \frac{1}{z}$ holomorph in $\C \setminus \{0\}$.
@@ -120,7 +120,7 @@ Sei $f : U \to V$ biholomorph, $z \in U$.
Dann ist $f'(z) \neq 0$ und für $w = f(z)$ gilt:
\vspace*{-2mm}
-$$(f^{-1})'(w) = \frac{1}{f'(f^{-1}(w))} = \frac{1}{f'(z)}$$
+\[ (f^{-1})'(w) = \frac{1}{f'(f^{-1}(w))} = \frac{1}{f'(z)} \]
Weiterhin existieren offene nichtleere $U \subseteq D$ mit $u_0 \in U, V \subseteq \C$ s.d. $\restrictedto{f}{U}$ biholomorph ist, wenn $f \in H(d) \cap C^1(D,\R^2)$, $z_0 \in D$ mit $f'(z_0) \neq 0$ gilt.
@@ -227,7 +227,7 @@ Geschrieben $f \in PC([a,b],\C)$.
Solche Funktionen sind integrierbar:
\vspace*{-4mm}
-$$\int_a^b f(t) dt := \int_a^b \text{Re } f(t) dt + i \int_a^b \text{Im } f(t) dt \in \C$$
+\[ \int_a^b f(t) dt := \int_a^b \text{Re } f(t) dt + i \int_a^b \text{Im } f(t) dt \in \C \]
\subsection*{Hauptsatz}
@@ -238,9 +238,9 @@ $\iff \text{Re } f, \text{Im } f$ besitzen Ableitungen in $\R$.
Ist $f$ auf $[a,b]$ diffbar und $g, f' \in C([a,b],\C)$. Dann gilt der Hauptsatz:
\vspace*{-3mm}
-$$\int_a^b f'(t) dt = f(b) - f(a)$$
+\[ \int_a^b f'(t) dt = f(b) - f(a) \]
-$$\exists \frac{d}{dt} \int_a^t g(s) ds = g(t) \text{ für } t \in [a,b]$$
+\[ \exists \frac{d}{dt} \int_a^t g(s) ds = g(t) \text{ für } t \in [a,b] \]
\subsection*{Kurven und Parametrisierungen}
@@ -257,7 +257,7 @@ $\gamma$ ist auch \emph{Parametrisierung} ihres Bildes $\Gamma$.
Sei $\gamma \in PC^1([a,b],\C)$ mit Bild $\Gamma = \gamma([a,b])$ und $f \in C(\Gamma,\C)$. Dann ist das \emph{komplexe Kurvenintegral}:
\vspace*{-2mm}
-$$\int_\gamma f dz = \int_\gamma f(z) dz := \int_a^b f(\gamma(t))\gamma'(t) dt$$
+\[ \int_\gamma f dz = \int_\gamma f(z) dz := \int_a^b f(\gamma(t))\gamma'(t) dt \]
Die Länge von $\gamma$ ist $l(\gamma) = \int_a^b |\gamma'(t)| dt$.
@@ -279,12 +279,12 @@ Sei $\gamma \in PC^1([a,b],\C)$ mit Bild $\Gamma$, $f_n, f \in C(\Gamma,\C)$ fü
$(f_n)$ konv. glm. auf $\Gamma$ gegen $f$
\vspace*{-2mm}
-$$\implies \displaystyle\lim_{n\to\infty} \int_\gamma f_n dz = \int_\gamma f dz$$
+\[ \implies \displaystyle\lim_{n\to\infty} \int_\gamma f_n dz = \int_\gamma f dz \]
$\sum_{n=1}^\infty f_n$ konv. glm. auf $\Gamma$
\vspace*{-2mm}
-$$\implies \displaystyle\sum_{n=1}^\infty \int_\gamma f_n dz = \int_\gamma \displaystyle\sum_{n=1}^\infty f_n dz$$
+\[ \implies \displaystyle\sum_{n=1}^\infty \int_\gamma f_n dz = \int_\gamma \displaystyle\sum_{n=1}^\infty f_n dz \]
\columnbreak
@@ -293,7 +293,7 @@ Abbildung $H : z \mapsto \int_\Gamma h(z,w) dw \in C(D,\C)$
$z \mapsto h(z,w) \in H(D)$ mit $\frac{\partial}{\partial z} h \in C(D \times \Gamma, \C)$
\vspace*{-2mm}
-$$\implies \frac{d}{dz} \int_\gamma h(z,w) dw = \int_\gamma \frac{\partial}{\partial z} h(z,w) dw$$
+\[ \implies \frac{d}{dz} \int_\gamma h(z,w) dw = \int_\gamma \frac{\partial}{\partial z} h(z,w) dw \]
d.h. $H$ ist holomorph mit dieser Ableitung.
@@ -324,14 +324,14 @@ Es ergibt sich für jeden stückweisen $C^1$-Weg in $D$: $\int_\gamma f dz = F(\
Seien $w_0 \in D, f \in C(D,\C) \cap H(D \setminus \{w_0\})$ und $\Delta \subseteq D$ ein abgeschlossenes Dreieck. Dann:
\vspace*{-2mm}
-$$\int_{\partial\Delta} f dz = 0$$
+\[ \int_{\partial\Delta} f dz = 0 \]
\subsection*{Cauchys Integralsatz}
Seien $D$ sternförmiges Gebiet, $f \in H(D)$ und $\gamma \in PC^1([a,b],D)$ geschlossen, dann gilt:
\vspace*{-2mm}
-$$\int_\gamma f dz = 0$$
+\[ \int_\gamma f dz = 0 \]
Dies gilt auch für $f \in C(D,\C) \cap H(D\setminus \{\omega_0\})$.
@@ -382,12 +382,12 @@ Für ganze $f$ gilt $R(z_0)=\infty$.
Sei $f : [0,\infty) \to \C$ messbar und $\exists M, \omega \geq 0 \forall t \geq 0 : |f(t)| \leq Me^{\omega t}$. Dann ex. die \emph{Laplacetransformation}:
\vspace*{-2mm}
-$$\hat f(\lambda) = \int_0^\infty e^{-\lambda t} f(t) dt \text{ für Re } \lambda > \omega$$
+\[ \hat f(\lambda) = \int_0^\infty e^{-\lambda t} f(t) dt \text{ für Re } \lambda > \omega \]
Diese ist auf $\{\lambda \in \C | \text{Re } \lambda > \omega\}$ holomorph mit:
\vspace*{-4mm}
-$$\hat f^{(n)}(\lambda) = (-1)^n \int_0^\infty e^{-\lambda t} t^n f(t) dt, \text{ Re } \lambda > \omega, n \in \N$$
+\[ \hat f^{(n)}(\lambda) = (-1)^n \int_0^\infty e^{-\lambda t} t^n f(t) dt, \text{ Re } \lambda > \omega, n \in \N \]
\subsection*{Satz von Morera}
@@ -452,7 +452,7 @@ Die \emph{Gammafunktion} $\Gamma(z) = \int_0^\infty t^{z-1}e^{-t} dt, Re \ z > 0
Sei $f \in H(D), B := B(z_0,r), r > 0, \overline B \subseteq D$. Weiter:
\vspace*{-3mm}
-$$0 \leq |f(z_0)| < \min_{x \in \partial B} |f(x)|$$
+\[ 0 \leq |f(z_0)| < \min_{x \in \partial B} |f(x)| \]
Dann hat $f$ Nullstelle in $B$.
@@ -499,7 +499,7 @@ Sind alle geschlossenen stückweisen $C^1$-Wege in $N$ nullhomotop, so heißt $N
Sei $D$ Gebiet, $f \in H(D), \gamma_0, \gamma_1$ auf $D$ homotope stückweise $C^1$-Wege. Dann:
-$$\int_{\gamma_0} f dz = \int_{\gamma_1} f dz$$
+\[ \int_{\gamma_0} f dz = \int_{\gamma_1} f dz \]
Insb. gilt $\int_{\gamma_0} f dz = 0$, wenn $\gamma_0 \sim_D 0$.
@@ -509,7 +509,7 @@ Cauchys Integralsatz gilt auf einfach zusammenhängenden Gebieten in $D$.
Sei $\overline B(z_0,r) \subseteq D, r > 0, z \in B(z_0,r), k(t) = z_0 + re^{it}$ für $t \in [0, 2\pi], n \in \N_0$ und $\gamma$ zu $k$ auf $D \setminus \{z\}$ homotoper stückweiser $C^1$-Weg. Dann:
-$$f^{(n)}(z) = \frac{n!}{2\pi i} \int_\gamma \frac{f(w)}{(w-z)^{n+1}} dw$$
+\[ f^{(n)}(z) = \frac{n!}{2\pi i} \int_\gamma \frac{f(w)}{(w-z)^{n+1}} dw \]
\section*{Isolierte Singularitäten}
@@ -565,7 +565,7 @@ Diese konvergiert, falls Grenzwerte in $\C$ ex.:
Ist dies der Fall, wird definiert:
\vspace*{-4mm}
-$$\sum_{n=-\infty}^{+\infty} a_n(z-c)^n := \sum_{n=0}^{+\infty} a_n(z-c)^n + \sum_{n=1}^{+\infty} a_{-n}(z-c)^{-n}$$
+\[ \sum_{n=-\infty}^{+\infty} a_n(z-c)^n := \sum_{n=0}^{+\infty} a_n(z-c)^n + \sum_{n=1}^{+\infty} a_{-n}(z-c)^{-n} \]
\subsubsection*{Satz von Laurent}
@@ -573,7 +573,7 @@ Seien $f \in H(D), n \in \Z, z_0 \in \C$ und $R > 0$ s.d. $D_0 := B(z_0,R) \setm
Für $r \in (0,R)$ sei $\gamma_r : [0,2\pi] \to \C, t \mapsto z_0 + re^{it}$ und:
-$$a_n := \frac{1}{2\pi i} \int_\gamma \frac{f(w)}{(w-z_0)^{n+1}} dw$$
+\[ a_n := \frac{1}{2\pi i} \int_\gamma \frac{f(w)}{(w-z_0)^{n+1}} dw \]
Diese Koeff. sind eindeutig und unabhg. $r \in (0,R)$.
@@ -597,7 +597,7 @@ Sei $z_0 \in \C$ isolierte Singularität von $f \in H(D)$ und $a_n$ Koeffiziente
Das \emph{Residuum} von $f$ bei $z_0$ ist definiert als:
-$$\text{Res}(f,z_0) := a_{-1} = \frac{1}{2\pi i} \int_{\partial B(z_0,r)} f(w) dw$$
+\[ \text{Res}(f,z_0) := a_{-1} = \frac{1}{2\pi i} \int_{\partial B(z_0,r)} f(w) dw \]
Hierbei gelte $\overline B(z_0,r) \setminus \{z_0\} \subseteq D$.
@@ -606,7 +606,7 @@ Hierbei gelte $\overline B(z_0,r) \setminus \{z_0\} \subseteq D$.
Seien $f \in H(D)$ und $z_1, \dots, z_n \in \C$ alle isolierten Singularitäten von $f$. Sei $p$ ein geschlossener, einfacher, positiv orientierter Polygonzug in $D$ mit Bild $P$ s.d. alle $z_j$ im von $P$ umschlossenen Gebiet $G$ liegen und $\overline G \setminus \{z_1,\dots,z_n\} \subseteq D$ ist. Weiterhin sei $\gamma \in PC^1([a,b],D)$ zu $p$ auf $D$ homotop. Dann:
\vspace*{-4mm}
-$$\int_\gamma f dz = 2\pi i \sum_{j=1}^n \text{Res}(f,z_j)$$
+\[ \int_\gamma f dz = 2\pi i \sum_{j=1}^n \text{Res}(f,z_j) \]
\subsubsection*{Residuen von Polen $m$-ter Ordnung}
@@ -621,13 +621,13 @@ Sei $z_0$ Pol $m$-ter Ordnung von $f \in H(D)$ und $g$ die holomorphe Fortsetzun
Insb. gilt also für $m=1$:
\vspace*{-2mm}
-$$\text{Res}(f,z_0) = \lim_{z \to z_0}(z-z_0)f(z)$$
+\[ \text{Res}(f,z_0) = \lim_{z \to z_0}(z-z_0)f(z) \]
\subsection*{Argumentprinzip}
Seien $f \in H(D), z_1, \dots, z_n \in D$ die Nullstellen von $f$ mit Ordnungen $m_1, \dots, m_n \in \N$ und $p$ geschlossener, einfacher, positiv orientierter Polygonzug in $\hat D := D \setminus \{z_1,\dots,z_n\}$ mit Bild $P$ s.d. die Nullstellen im von $P$ umschlossenen Gebiet $G$ liegen mit $\overline G \subseteq D$. Sei $\gamma$ in $\hat D$ zu $p$ homotope, geschlossene stückweise $C^1$-Kurve. Dann:
-$$\frac{1}{2\pi i} \int_\gamma \frac{f'(z)}{f(z)} dz = \sum_{j=1}^n m_j$$
+\[ \frac{1}{2\pi i} \int_\gamma \frac{f'(z)}{f(z)} dz = \sum_{j=1}^n m_j \]
\subsubsection*{Satz von Rouché}
@@ -636,6 +636,6 @@ Sei $f \in H(D), z_j \in \C, m_j \in \N$ und Weg $\gamma$ mit Bild $\Gamma$ ents
Seien $\omega_1, \dots, \omega_\nu$ Nullstellen von $g$ im von $\gamma$ umschlossenen Gebiet mit Vielfachheiten $\mu_k \in \N$:
\vspace*{-2mm}
-$$\sum_{j=1}^n m_j = \sum_{k=1}^\nu \mu_k$$
+\[ \sum_{j=1}^n m_j = \sum_{k=1}^\nu \mu_k \]
d.h. die Summe der Nullstellenordnungen von $f$ ist gleich der Summe der Vielfachheiten von $g$.
diff --git a/content/graph_theory.tex b/content/graph_theory.tex
index de31f61..961b5cf 100644
--- a/content/graph_theory.tex
+++ b/content/graph_theory.tex
@@ -58,7 +58,7 @@ $\text{rad}(G) := \displaystyle\min_{u \in V(G)} \max_{v \in V(G)} d(u,v)$ is th
\subsection*{Handshake Lemma}
-$$2|E| = \sum_{v \in V} d(v)$$
+\[ 2|E| = \sum_{v \in V} d(v) \]
Furthermore the sum of all degrees is even and thus \#vertices with odd degree is also even.
@@ -129,7 +129,7 @@ $G$ is $k$-connected if $k-1$ vertices can be removed without disconnecting.
Let $\kappa'(G)$ be the edge-connectivity of $G$:
\vspace*{-3mm}
-$$\kappa(G) \leq \kappa'(G) \leq \delta(G)$$
+\[ \kappa(G) \leq \kappa'(G) \leq \delta(G) \]
\subsection*{Menger's Theorem}
@@ -247,7 +247,7 @@ Every planar graph is $5$-choosable.
\subsection*{Greedy chromatic number estimate}
-$$\chi(G) \leq \Delta(G)+1$$
+\[ \chi(G) \leq \Delta(G)+1 \]
\subsection*{Brook's Theorem}
@@ -255,18 +255,18 @@ If $G$ is connected and neither complete nor an odd cycle then $\chi(G) \leq \De
\subsection*{König's Theorem}
-$$G \text{ bipartite} \implies \chi'(G) = \Delta(G)$$
+\[ G \text{ bipartite} \implies \chi'(G) = \Delta(G) \]
\subsection*{Vizing's Theorem}
-$$\chi'(G) \in \{\Delta(G), \Delta(G)+1\}$$
+\[ \chi'(G) \in \{\Delta(G), \Delta(G)+1\} \]
\section*{Extremal Graph Theory}
For $n \in \N$ and graph $H$ the \emph{extremal number} $ex(n,H)$ is the max number of edges in a graph of order $n$ s.t. it doesn't contain subgraph $H$.
\vspace*{-3mm}
-$$ex(n,H):=\max\{\|G\| : |G|=n, H \not\subseteq G\}$$
+\[ ex(n,H):=\max\{\|G\| : |G|=n, H \not\subseteq G\} \]
Correspondingly $EX(n,H$ is the set of graphs on $n$ vertices and $ex(n,H)$ edges that are $H$-free.
@@ -290,23 +290,23 @@ $T(n,r)$ doesn't contain $K_{r+1}$ and $t(n,r) := \|T(n,r)\|$.
For $r | n \ T(n,r)$ is denoted by $K_r^s$ where $n=r \cdot s$.
-$$t(n,r) = t(n-r,r)+(n-r)(r-1)+{r \choose 2}$$
+\[ t(n,r) = t(n-r,r)+(n-r)(r-1)+{r \choose 2} \]
\spacing
Comparing the number of edges in $K_n$ and $T(n,r)$:
-$$\lim_{n \to \infty} \frac{t(n,r)}{{n \choose 2}} = \left( 1-\frac{1}{r} \right)$$
+\[ \lim_{n \to \infty} \frac{t(n,r)}{{n \choose 2}} = \left( 1-\frac{1}{r} \right) \]
\subsubsection*{Tur\'{a}n's Theorem}
-$$EX(n,K_r) = \{T(n,r-1)\}$$
+\[ EX(n,K_r) = \{T(n,r-1)\} \]
\subsection*{$\epsilon$-regularity}
Let $X, Y \subseteq V(G)$ be disjoint and $\|X,Y\|$ is the number of edges between $X$ and $Y$.
-$$d(X,Y) := \frac{\|X,Y\|}{|X||Y|} \text{ is the density of $(X, Y)$}$$
+\[ d(X,Y) := \frac{\|X,Y\|}{|X||Y|} \text{ is the density of $(X, Y)$} \]
$\forall \epsilon > 0 : (X,Y)$ is an \emph{$\epsilon$-regular pair} if:
@@ -331,13 +331,13 @@ $\forall \epsilon > 0, m \geq 1 \exists M \in \N$ s.t. every graph $G$ with $|G|
$\forall r > s \geq 1, \epsilon > 0 \exists n_0 \in \N$ s.t. all graphs with $|V(G)| =: n \geq n_0$ vertices and
\vspace*{-2mm}
-$$|E(G)| \geq t_{r-1}(n)+\epsilon n^2$$
+\[ |E(G)| \geq t_{r-1}(n)+\epsilon n^2 \]
contain $K_r^s$ as a subgraph.
\subsection*{Asymptotic extremal number}
-$$\lim_{n \to \infty} \frac{ex(n,H)}{{n \choose 2}} = \frac{\chi(H)-2}{\chi(H)-1}$$
+\[ \lim_{n \to \infty} \frac{ex(n,H)}{{n \choose 2}} = \frac{\chi(H)-2}{\chi(H)-1} \]
e.g. $ex(n,K_5 \setminus \{e\}) \simeq \frac{2}{3} \cdot {n \choose 2}$ as $\chi(K_5 \setminus \{e\}) = 4$.
@@ -352,7 +352,7 @@ $z(m,n;s,t) \leq (s-1)^{1/t}(n-t+1)m^{1-1/t}+(t-1)m$
i.e. for $m=n$ and $t=s$:
\vspace*{-2mm}
-$$z(n,n;t,t) \in \mathcal{O}(n^{2-\frac{1}{t}})$$
+\[ z(n,n;t,t) \in \mathcal{O}(n^{2-\frac{1}{t}}) \]
\section*{Ramsey Theory}
@@ -381,7 +381,7 @@ e.g. $R(2,k) = R(k,2) = k$ and $R(3) = 6$
\subsection*{Ramsey Theorem}
\vspace*{-2mm}
-$$\forall k \in \N : \sqrt{2}^k \leq R(k) \leq 4^k$$
+\[ \forall k \in \N : \sqrt{2}^k \leq R(k) \leq 4^k \]
Particularly Ramsey, asymmetric Ramsey and graph Ramsey numbers are finite.
@@ -473,7 +473,7 @@ A \emph{property} $\mathcal{P}$ is a set of graphs.
$G \in \mathcal{G}(n,p_n)$ \emph{almost always} has property $\mathcal{P}$ if:
\vspace*{-2mm}
-$$\mathbb{P}(G \in \mathcal{G}(n,p_n) \cap \mathcal{P}) \to 1 (n \to \infty)$$
+\[ \mathbb{P}(G \in \mathcal{G}(n,p_n) \cap \mathcal{P}) \to 1 (n \to \infty) \]
If $p_n$ is constant it is said that \emph{almost all} $G \in \mathcal{G}(n,p)$ have property $\mathcal{P}$.
@@ -493,7 +493,7 @@ Such functions do not exist for all properties $\mathcal{P}$.
Let $G$ be a graph on $n$ vertices and $m$ edges.
\vspace*{-2mm}
-$$\mathbb{P}(G = \mathcal{G}(n,p)) = p^m (1-p)^{{n \choose 2}-m}$$
+\[ \mathbb{P}(G = \mathcal{G}(n,p)) = p^m (1-p)^{{n \choose 2}-m} \]
\subsection*{Expected values and indicators}
@@ -502,17 +502,17 @@ Let $X : \mathcal{G}(n,p) \to \N$ be a random variable.
When $X$ counts e.g. the nu